subject
Mathematics, 08.12.2020 14:00 cyndiann2002

Whould anyone mind helping please. The first box says 6.25x+12x=30.75 the second box says 6.25x+12=30.75 3rd box says 6.25x+30.75=12 the 4th box says 30.75+12=6.75x . the last drop down says how many people went to the movies 2,3,4,5 .


Whould anyone mind helping please. The first box says 6.25x+12x=30.75 the second box says 6.25x+12=

ansver
Answers: 2

Another question on Mathematics

question
Mathematics, 21.06.2019 13:30
Which is the product of 58 and 1,000? a. 0.058 b. 5,800 c. 58,000 d. 580,000
Answers: 1
question
Mathematics, 21.06.2019 22:30
For the chance to be team captain, the numbers 1-30 are put in a hat and you get two chances to pick a number, without replacement. which formula correctly shows how to find the probability that you choose the number 1 and then 2?
Answers: 1
question
Mathematics, 22.06.2019 00:50
After reading 80% of her e-mails in her inbox, danette still has m unread e-mails. which of the following expressions could represent the number of e-mails danette had in her inbox before she started reading? choose 2 answers: (choice a) 5m (choice b) 1.8m (choice c) 80m (choice d) m/0.8 (choice e) m/1βˆ’0.8
Answers: 1
question
Mathematics, 22.06.2019 02:00
If p(x) is the total value of the production when there are x workers in a plant, then the average productivity of the workforce at the plant is a(x) = p(x) x . (a) find a'(x). a'(x) = xp'(x) βˆ’ p(x) x a'(x) = xp'(x) βˆ’ p(x) x2 a'(x) = p'(x) βˆ’ p(x) x a'(x) = xp'(x) βˆ’ p'(x) x2 a'(x) = p'(x) βˆ’ xp(x) x2 why does the company want to hire more workers if a'(x) > 0? a'(x) > 0 β‡’ a(x) is ; that is, the average productivity as the size of the workforce increases. (b) if p'(x) is greater than the average productivity, which of the following must be true? p'(x) βˆ’ xp(x) > 0 p'(x) βˆ’ xp(x) < 0 xp'(x) βˆ’ p'(x) > 0 xp'(x) βˆ’ p(x) < 0 xp'(x) βˆ’ p(x) > 0
Answers: 2
You know the right answer?
Whould anyone mind helping please. The first box says 6.25x+12x=30.75 the second box says 6.25x+12=3...
Questions
Questions on the website: 13722362